Вы находитесь на странице: 1из 16

sequence

is bounded, and

. Prove that

exists, and determine its value Start with the case Suppose a subsequence Then Repeat the argument to get etc. Thus there are arbitrarily large subseqential limits of . This violates the boundedness of Therefore the only subseqential limit of can be which is equivalent to saying If apply the above to to see Let be real numbers. The sequence Find Let , were is 1.. Let a sequence defined by . Prove that . satisfies for all . Then Therefore we can chose by Because had limit if

the limit of

Let . Substitute this into the given recurrence relation, then

By (A),

As

is bounded upward(why??), we get

as

. Thus

, and this gives

Therefore Let a sequence defined by is finite. Direct computation gives and . and . Prove that

Let

be such that

and

. Then

and . Since the LHS is a degree-6 polynomial in and the RHS a degee-7 polynomial in , we must have LHS < RHS for all sufficiently large

in particular for all

. Hence, by induction,

for all

and since The sequence

exists, so does . is defined as follow

We define the sequence

as follow:

Prove that the sequence

has a limit and find it.

Put

to get

The characteristic equation is

Thus

Plugging

and

in

and solving the system we get

Now Consider

a set of positive real numbers such that

Evaluate: . The given inequality is equivalent to By (A),

So is bounded. Therefore, by Bolzano-Weierstrauss theorem, there exist an infinite subsequence of which is convergent; Let it , and let its limit . Note that also satisfies (A). Applying squeezing theorem with (A), we get

Thus Therefore

. This result gives . , given by , prove that : . is increasing on . We have

The squence satisfies Notice the function . Let us prove induction for

. We have

. Then, since .

for

, by

But then proven to be true. Therefore the sequence

, since is equivalent to is increasing, and bounded above by (since . It remains to show .

). It follows it is convergent to some limit Given : , . Prove that exists a limit of and find this limit. Put , which yields

i.e. a system

Plugging

into

we get

The characteristic equation is

, thus there exist constants

such that

, and is not zero as the sequence is obviously not constant, since

Therefore

, yielding

Obviously,

is convergent and its limit is : and . Find (with ).

Given a sequence

No, unfortunately things are not getting simpler by that observation. The following proves the fact that being of paramout importance, since for we have

1.

for

. But if

then

for all

, and so

. Moreover, the same can be said if the sequence is bounded. We will assume therefore in the sequel that for all , with the sequence being unbounded.

2. If

then , it follows the sequence

; since , it means this is true. Denoting has positive terms.

3. If

it follows

, since the roots of

are , and so

, with and . But then the sequence is shown to ultimately being monotone decreasing, thus bounded, absurd. 4. We are then left with the case recurrence relation for all . This means

for all . But we have the

leading to . For large enough we will then have , absurd. If is a positive real number, consider the sequence defined by:

Show that there exist a real number i.) for all ii.) for all I claim the sequence

such that:

the sequence . We need to show that conditions (i) and (ii) are satisfied.

Condition (i). If also larger, so still

, then .

for all

, so

. If

is larger, then all subsequent

are

Condition (ii). Suppose that, for

, we do not have

. . Now . . Continuing this reasoning, we eventually for

Suppose for sake of contradiction there is an so that ; then all , so , a contradiction. Thus no satisfies If then deduce that , and

and this condition holds for all

. It is also clear (from the identity

) that

for all

. In the same way, we have

for all

. It is clear that . We have

for all , and that

is strictly increasing; we will now show that

and it is clear this tends to as . Because desired. Let Find , we must have

. In the above manipulation we make repeated use of the identity

. As a result, if

, then we do in fact have

, as

be a sequence of real numbers with

and

for any

We first prove that We claim that assume justified. Then for

as

. and , thus this is satisfied for and . Now

for all . Indeed, . Then

and by mathematical induction, our claim is

for all

, thus the conclusion follows from the squeezing lemma.

This shows that

Therefore by Cesaro-Stolz theorem,

Given a sequence Prove that

such that

By Cezaro-Stolz lemma Hence Let

, , such that for all

. , i.e. for . This means for all . . Then . But then since and

; there exists

and and

, it means there exists for all If and for

such that

. This is enough. , prove that converges to , and letting

So I've tried using that if Then 1. First of all, 2. If sequence. 3. If sequence. 4. If sequence.

converges to , then converges to should converge to for all nonnegative integers . , then

is a bounded decreasing sequence and

is a bounded increasing

, then

is a bounded decreasing sequence and

is a bounded increasing

, then

is a bounded increasing sequence and

is a bounded decreasing

The rest of your reasoning is correct, including the monotonically decreasing nature of the sequence I believe there's a neater way to tie this up, but I haven't found it yet. 2. Let x be a positive no. A sequence of real numbers is defined as follows : , for all Now, (a) Show that for all (b) Hence find , .For (a), I assume the problem was . ,...... or in general

We apply induction taking So base case is done for Let us assume the statement to be true for Now

So, by theory of induction, Applying this we can solve part (b) easily.

for all nonnegative integers .

As

3. Let

and

for

Prove 4. The following AM-GM consequence may be useful:

so that

for all .

5. For we have (The first inequality is easy; for the second, the difference between the two sides is , and this is positive if and only if . So, if we ever get small, we stay that way. Edit: okay, I think this idea can be modified reasonably easily to finish: choose large (say that (With 6. Let . Then it follows from a similar argument (replacing 5 with ) that , the difference we need to estimate is positive for any .) and for for all ) so . .) Thus, if then

find It is obvious that the sequence . Thus, .

is decreasing . By induction one can easily prove that :

and by passing to limit in the recurrence relation we get :

i.e.

On the other hand, Accordingly, by the Stolz-Cesaro theorem we obtain that : . Hence,

. But :

. Therefore,

using again the Stolz-Cesaro theorem we obtain the required limit : and be positive integers such that .

.Let

Denote by

the number of

such that , then for each , find

has

distinct real solutions.

Determine the value of such that 7. For , we obviously have For If If , then : :

is the number of pairs

such that

and so :

So

So

So

So

when

and

when

And the required limit 8. Let and

when , for

and when

find The recurrence relation is equivalent to be increasing, with . 1. We have . Therefore 2. We have and some large enough for all , hence . for all large enough , hence . Thus .

. The sequence is thus seen to

for all . Thus

for all . Since

. Therefore

was arbitrarily chosen, it follows 3. Together, the two facts above imply written as : and

.As noted above, the recurrence relation can be .

On the other hand, we have that :

, so

But : Therefore, by the Stolz-Cesaro theorem we obtain that : 9. Let be the sequence with and tends to infinity. Easy to show that strictly decreasing. It follows from Let Hence that . . Find the limit of so there exists .

when

But

So

So answer is 10. is strictly increasing and continuous function such that . Show that for every real :

and

11. First, we will prove by Mathematical Induction in , that . For , it is obvious that it is true. Supposed that the statement is true for some , then we will prove that the statement is true for . Because the statement is true for , then we have that , then and we are done. We also know that is a surjective function. Supposed that , then when is a positive integer and while is a negative integer. Let be any real number, then we can choose natural number such that , by the continuity, we are done. Note also that for any integer (it can be proved by using Mathematical Induction in ), and moreover for any integers and . Now, if is in the form mod , then for some integer , and we know that there exist such that , by the surjectivity of the function. Then by the monotonicity, and

If is in the form mod , then

for some integer ,

and we know that there exist

such that

by the surjectivity of the function. Then, by monotonicity, and

At last, when is in the form mod , then and

for some integer

and we know that

Let If

From the description above, we know that, while tends to infinity, we have that using squeezed theorem. . Find . , then , and furthermore , , i.e. , . , it means , for all and , but if , it means the sequence for , it only means is divergent. ; since , . for all , so for . In the sequel . , of roots and .

by

Consider ; so , The characteristic polynomial is So , where Therefore Now, if while Let

Find

and so

This series is obviously convergent and its limit is

Hence the result : 12. Let . Define the sequence for each 13. Better write it as

by Show that . Clearly is convergent. , so the sequence is non-decreasing. for all . . In the sequel

It is therefore enough to prove it is upper bounded, and then If for all we're done, so assume there exists some so that we will always work with , for some conveniently large .

Then

. Iterating this we get

. Let us

estimate

For a sequence induction on

of sub-unitary positive real numbers, we have , since it is trivially true for , and

by

. Applying this to our case, we get means 14. Let Find 15. Let Consider the sequence It's easy to see (linear sequences) that with and and is a convergent sequence whose limit is . ; and , and we're done. with for large enough . This

Consider the sequence It's easy to see with induction (and using convergent sequence whose limit is .

with

and ) that and so that is a

Consider the sequence

with

and is a convergent

Then and it's easy to see that and so that sequence whose limit is and so that is a convergent sequence whose limit is .

Back to original sequence, we get Choosing then , we get that

and is a convergent sequence whose limit is

And so Let and let the sequence be defined as follows :

and Prove that

for all converges to a real number has one solution since and that for all , is strictly increasing.

also, narrowing it down and so

so the zero

its easy to see that

and so thus, we must have also, alternate on each side of and since from our above calculation, so we have

16. A quick way for the first part is by the Banach Fixed point theorem.

is a

contraction mapping since for , , so there exists a unique fixed point of , which is the limit of the sequence. It's easy to check by substitution that the end-points can't be this limit. Let be the sequence such that is strictly increasing on proof: we have: , . and Find lemma:

we have to deal now with two cases: if if then then and it's done. because : wich is true,so we are done. the mean idea: i evaluated some values taken by the sequence,and noticed that it approches .however,our sequence oscillates around the value ,so the straigthforward way that consist on showing that it's increasing (decreasing) and majorized (minorized) won't work here.then i had the idea to majorize the sequence by an other positive sequence whom decreases and converges to (and the more slowly she goes to ,the more possible that it will work). i opted for something of the form , now,it's all about to find so that the double-step induction works,and it's not a probleme if it doesn't work for the first terms. double-step induction: we will supose that : basis). we should have : the left side's inequality holds for all positive integers ,otherwise,the right side's one is equivalent to: ok! i want my propriety to work starting from , we should have then: .unfortunately,such doesn't exist because: (check it). it doesn't work when starting from too.but,what's about ? let's see, we are searching for such that : , , and , and , , for some positive integer and try to find some such that:

(the induction step) , and our should also work for two consecutive positive integers (as a

is an increasing sequence(using the lemma at the top of the post),so it will suffice to have:

. .

wich is equivalent to: synthesis: we can choose I just choosed remark: Now that it works for

, so that we will get:

,it will also work for

.so:

I choosed 17. Note that

, because i wished if you follow my traces you will notice it Let for all . Then find . is not well-defined unless some additional conditions are added. is a sequence satisfying given recurrence relation, and put . In particular, we have

, and assume

To see this, suppose

. Then .

the recurrence relation reduces to

But it is easy to see that becomes densier as grows, so that also becomes densier in . In particular, for any cleverly chosen will make converge to . One way to avoid this absurdity is to impose the condition . Indeed, for there exists a unique solution This recurrence relation yields . on . , therefore we have

Вам также может понравиться